Solving IVP using Laplace Transforms

Click For Summary
The discussion focuses on solving the initial value problem x'' + x = cos(3t) using Laplace transforms, with initial conditions x(0) = 1 and x'(0) = 0. The solution process involves applying the Laplace transform to the differential equation, leading to the expression for X(s) in terms of partial fractions. Participants clarify that there is no need for a common denominator and guide through the setup for partial fraction decomposition. The final solution is derived as x(t) = (1/8)(9cos(t) - cos(3t)). This method effectively demonstrates the application of Laplace transforms in solving differential equations.
daemon_dkm
Messages
6
Reaction score
0

Homework Statement



"Use Laplace transforms to solve the initial value problem" from section 4.2 in Elementary Differential Equations (6th ed.) Edwards & Penny

x'' + x = cos3t; x(0) = 1 & x'(0) = 0


Homework Equations



L{coskt} = s/(s²+k²)
Apparently the answer is 1/8[9cos(t)-cos(3t)].

The Attempt at a Solution




x'' + x = cos3t
L{x''} = s²*X(s) - s*1 - 0
s²X(s) - s + X(s) = s/(s²+3²)
X(s)(s²+1) = s/(s²+3²) + s
X(s) = s/[(s²+3²)(s²+1)] + s/(s²+1)

This is where I get stuck. Do I get a common denominator then do partial fractions?
 
Physics news on Phys.org
There's no need to get a common denominator. Just use partial fractions to break up the first term.
 
What do I do with the right most part then?
Would I set it up like
s/[(s²+3²)(s²+1)] + s/(s²+1)
= (As+B)/(s²+3²) + (Cs+D)/(s²+1) + s/(s²+1)
s = (As+B)(s²+1) +(Cs+D)(s²+3) + s(s²+3²)
 
You can ignore the last part for now, so you want to start with

\frac{s}{(s^3+3^2)(s^2+1)} = \frac{As+B}{s^2+3^2}+\frac{Cs+D}{s^2+1}

Then

X(s) = \frac{As+B}{s^2+3^2}+\frac{Cs+D}{s^2+1} + \frac{s}{s^2+1}
 
Thank you! That's just what I needed to know!

Because then

0s3 = A3 + C3 => A = -C
0 = B2+D2
0 = B + 3D
s = As + 9Cs
1 = -C + 9C
1 = 8C => C = 1/8 so A = -1/8
<br /> X(s) = -\frac{1}{8}\frac{s}{s^2+3^2}+\frac{1}{8}\frac{s}{s^2+1} + \frac{8}{8}\frac{s}{s^2+1}<br />

<br /> x(t) = L{X(s)} = -\frac{1}{8}\cos(3t)+\frac{9}{8}\cos(t)<br />
 
Question: A clock's minute hand has length 4 and its hour hand has length 3. What is the distance between the tips at the moment when it is increasing most rapidly?(Putnam Exam Question) Answer: Making assumption that both the hands moves at constant angular velocities, the answer is ## \sqrt{7} .## But don't you think this assumption is somewhat doubtful and wrong?

Similar threads

  • · Replies 7 ·
Replies
7
Views
2K
Replies
9
Views
2K
  • · Replies 1 ·
Replies
1
Views
999
  • · Replies 1 ·
Replies
1
Views
1K
  • · Replies 3 ·
Replies
3
Views
3K
  • · Replies 8 ·
Replies
8
Views
2K
  • · Replies 8 ·
Replies
8
Views
2K
  • · Replies 3 ·
Replies
3
Views
1K
  • · Replies 26 ·
Replies
26
Views
3K
  • · Replies 14 ·
Replies
14
Views
3K